$M = (M_t)_{t\geq 0}$ obtained by Itô’s formula is a martingale

martingalesstochastic-calculusstochastic-differential-equationsstochastic-integralsstochastic-processes

Here I defined a non-negative stochastic process. Now, taking $F(t,x) = tx^2$, I would like to find that the continuous local martingale “part'' obtained by Itô's formula is indeed a Martingale. To do so, I obtained, by applying Itô's formula, and
$$
d X_t = 3dt + 2\sqrt{X_t}d B_t, \quad\quad d\langle X,X\rangle_t = 4X_t\langle B,B\rangle_t,
$$

the following formula:
$$
F(t,X_t) = \int_0^2 [X_s^2 + 10s X_s ] ds + 4\int_0^t sX_s \sqrt{X_s}d B_s.
$$

Hence, I defined $M = (M_t)_{t\geq 0}$ by
$$
\int_0^t sX_s \sqrt{X_s}d B_s.
$$

To see that it is a martingale, it is sufficient to see that $E\sqrt{\langle M,M\rangle_s}< \infty$, but, by (concave function version) Jensen's inequality, $E\sqrt{\langle M,M\rangle_s}\leq \sqrt{E\langle M,M\rangle_s}$, and then it is enough to see that $E\langle M,M\rangle_s<\infty$. Hence,
$$
E\langle M,M\rangle_s = E\int_0^t s^2 X_s^3 ds.
$$

Since $X_s\geq 0$, we can apply Fubini's theorem, and hence
$$
E\int_0^t s^2 X_s^3 d s = \int_0^t s^2 EX_s^3 ds.
$$

My question is: How can I asssure that $EX_s^3 <\infty$? Is there any other way of doing it? Thank you in advance and I hope you find it interesting.

Best Answer

There are general statements on the existence of moments of solutions to SDEs. If you are lucky, you have one of those results at your disposal to deduce that $\sup_{s \leq t} \mathbb{E}(|X_s|^3)<\infty$ for all $t>0$. If this is not the case, then you need to do some further calculations.

In the following, I will consider $\mathbb{E}(X_t^4)$ (rather than $\mathbb{E}(|X_t|^3)$) since this saves us the trouble to deal with the modulus. By Itô's formula, we have

$$X_t^4 = 8 \int_0^t X_s^3 \sqrt{X_s} \, dB_s +30 \int_0^t X_s^3 \, ds.$$

Now we want to take the expectation, but since we do not yet know whether the expectation is finite, we first need to some stopping. To this end, define

$$\tau_r := \inf\{t \geq 0; |X_t| \geq r\},$$

note that $\tau_r \uparrow \infty$ as $r \to \infty$ and $|X_{t \wedge \tau_r}| \leq r$. Since

$$t \mapsto \int_0^{t \wedge \tau_r} X_s \sqrt{X_s} \, dB_s$$

is a martingale (because of the stopping!), and, hence, has expectatio zero, we get

$$\mathbb{E}(X_{t \wedge \tau_r}^4) = 30 \mathbb{E} \left( \int_0^{t \wedge \tau_r} X_s^3 \, ds \right).$$

Using the elementary estimate

$$x^3 \leq |x|^3 \leq 1+x^4, \qquad x \in \mathbb{R},$$

for $x=X_s$, we find that

$$\mathbb{E}(X_{t \wedge \tau_r}^4) \leq 30 \mathbb{E} \left( \int_0^{t \wedge \tau_r} (1+X_s^4) \, ds \right) \leq 30 \int_0^t \mathbb{E}(1+X_{s \wedge \tau_r}^4) \, ds.$$

If we define $u(t) := \mathbb{E}(1+X_{t \wedge \tau_r}^4)$ for fixed $r>0$, then

$$u(t) \leq 1+ 30 \int_0^t u(s) \, ds$$

and so Gronwall's lemma yields

$$u(t) \leq c e^{Mt}$$

for suitable constants $c,M>0$, which do not depend on $r>0$. Consequently, we have shown that

$$\mathbb{E}(1+X_{t \wedge \tau_r}^4) \leq c e^{Mt}, \qquad t \geq 0.$$

By Fatou's lemma, this implies

$$\mathbb{E}(1+X_{t}^4) \leq c e^{Mt}, \qquad t \geq 0.$$

In particular,

$$\sup_{t \leq T} \mathbb{E}(X_t^4) < \infty, \qquad T>0,$$

which also yields

$$\sup_{t \leq T} \mathbb{E}(X_t^3) < \infty, \qquad T>0.$$